Difference between revisions of "2025 USAJMO Problems/Problem 4"
Cooljoseph (talk | contribs) (→Solution: Edited cyc to sym in sum) |
(→Solution 2) |
||
(One intermediate revision by the same user not shown) | |||
Line 4: | Line 4: | ||
Let <math>n</math> be a positive integer, and let <math>a_0,\,a_1,\dots,\,a_n</math> be nonnegative integers such that <math>a_0\ge a_1\ge \dots\ge a_n.</math> Prove that<cmath>\sum_{i=0}^n i\binom{a_i}{2}\le\frac{1}{2}\binom{a_0+a_1+\dots+a_n}{2}.</cmath>Note: <math>\binom{k}{2}=\frac{k(k-1)}{2}</math> for all nonnegative integers <math>k</math>. | Let <math>n</math> be a positive integer, and let <math>a_0,\,a_1,\dots,\,a_n</math> be nonnegative integers such that <math>a_0\ge a_1\ge \dots\ge a_n.</math> Prove that<cmath>\sum_{i=0}^n i\binom{a_i}{2}\le\frac{1}{2}\binom{a_0+a_1+\dots+a_n}{2}.</cmath>Note: <math>\binom{k}{2}=\frac{k(k-1)}{2}</math> for all nonnegative integers <math>k</math>. | ||
− | == Solution == | + | == Solution 1 == |
By Vandermonde's, | By Vandermonde's, | ||
Line 10: | Line 10: | ||
~rhydon516 (sol credits to leo) | ~rhydon516 (sol credits to leo) | ||
+ | |||
+ | == Solution 2 == | ||
+ | |||
+ | We proceed by induction. | ||
+ | |||
+ | Base case: <math>n=1</math>. Note that <math>\frac{1}{2}\binom{a_0+a_1}{2} \ge \frac{1}{2}\binom{a_1+a_1}{2} = 2a_1(2a_1-1)/4 \ge a_1(a_1-1)/2 = \binom{a_1}2</math> | ||
+ | |||
+ | Inductive Hypothesis: Assume <math>\sum_{i=0}^n i\binom{a_i}{2}\le\frac{1}{2}\binom{a_0+a_1+\dots+a_n}{2}</math> | ||
+ | |||
+ | Inductive Step: We try to show it works for <math>n+1</math>: <math>\sum_{i=0}^{n+1} i\binom{a_i}{2}\le\frac{1}{2}\binom{a_0+a_1+\dots+a_{n+1}}{2}</math> | ||
+ | |||
+ | By the Inductive Hypothesis all we need to show is <math>(n+1)\binom{a_{n+1}}{2}\le \frac{1}{2}\binom{a_0+a_1+\dots+a_{n+1}}{2}-\frac{1}{2}\binom{a_0+a_1+\dots+a_{n}}{2}</math> | ||
+ | |||
+ | Let <math>s=a_0+a_1+..+a_n</math> and <math>c=a_{n+1}</math>. Now we need <math>(n+1)\binom{c}{2}\le \frac{1}{2}\binom{s+c}{2}-\frac{1}{2}\binom{s}{2}</math>. The LHS is <math>\frac{1}{2}(n+1)(c)(c-1)</math> the RHS is <math>\frac{1}{4}(2sc+c^2-c)</math>. If <math>c=0</math> we're done, otherwise we divide by <math>c</math> and multiply by <math>4</math> to get <math>2(n+1)(c-1)=2nc+2c-2n-2</math> which we are trying to show is <math>\le 2s+c-1</math>. Note that <math>s\ge (n+1)c</math> by the problem statement condition. So it simplifies to <math>2nc+2c-2n-2 \le 2nc+3c-1</math> which is obvious. | ||
==See Also== | ==See Also== |
Latest revision as of 16:54, 7 August 2025
Contents
Problem
Let be a positive integer, and let
be nonnegative integers such that
Prove that
Note:
for all nonnegative integers
.
Solution 1
By Vandermonde's,
with equality at
and
for
~rhydon516 (sol credits to leo)
Solution 2
We proceed by induction.
Base case: . Note that
Inductive Hypothesis: Assume
Inductive Step: We try to show it works for :
By the Inductive Hypothesis all we need to show is
Let and
. Now we need
. The LHS is
the RHS is
. If
we're done, otherwise we divide by
and multiply by
to get
which we are trying to show is
. Note that
by the problem statement condition. So it simplifies to
which is obvious.
See Also
https://artofproblemsolving.com/community/c5h3532101_bombardiro_crocodilo_vs_tralalero_tralala
2025 USAJMO (Problems • Resources) | ||
Preceded by Problem 3 |
Followed by Problem 5 | |
1 • 2 • 3 • 4 • 5 • 6 | ||
All USAJMO Problems and Solutions |
These problems are copyrighted © by the Mathematical Association of America, as part of the American Mathematics Competitions.